K
Khách

Hãy nhập câu hỏi của bạn vào đây, nếu là tài khoản VIP, bạn sẽ được ưu tiên trả lời.

3 tháng 7 2018

( 7.3 + 8.13) : ( 9/2/3 - y) = 39

( 21 + 104) : ( 29/3 - y) = 39

125 : ( 29/3-y) = 39

29/3-y = 125 - 39

29/3-y = 86

y = 29/3 -86

y = -229/3

3 tháng 7 2018

9.271025641

23 tháng 6 2015

1, \(\frac{1}{2}-\left(6\frac{5}{9}+x-\frac{117}{8}\right):\left(12\frac{1}{9}\right)=0\)

   \(\left(\frac{6.9+5}{9}+x-\frac{117}{8}\right):\frac{12.9+1}{9}=\frac{1}{2}\)

 ( . là nhân nha) 

    \(\left(\frac{59}{9}-\frac{117}{8}+x\right):\frac{109}{9}=\frac{1}{2}\)

    \(\frac{59}{9}-\frac{117}{8}+x=\frac{1}{2}\cdot\frac{109}{9}\)

    \(\frac{59}{9}-\frac{117}{8}+x=\frac{109}{18}\)

   \(x=\frac{109}{18}-\frac{59}{9}+\frac{117}{8}\)

\(x=\frac{113}{8}\)

23 tháng 6 2015

\(\left(y+\frac{1}{3}\right)+\left(y+\frac{2}{9}\right)+\left(y+\frac{1}{27}\right)+\left(y+\frac{1}{81}\right)=\frac{56}{81}\)

   \(y+\frac{1}{3}+y+\frac{2}{9}+y+\frac{1}{27}+y+\frac{1}{81}=\frac{56}{81}\)

\(4y+\frac{1}{3}+\frac{2}{9}+\frac{1}{27}+\frac{1}{81}=\frac{56}{81}\)

\(4y+\frac{49}{81}=\frac{56}{81}\)

\(4y=\frac{7}{81}\)

y      =  7/81:4

y       = 7/324

5 tháng 4 2016

theo bài ra ta có 
n = 8a +7=31b +28 
=> (n-7)/8 = a 
b= (n-28)/31 
a - 4b = (-n +679)/248 = (-n +183)/248 + 2 
vì a ,4b nguyên nên a-4b nguyên => (-n +183)/248 nguyên 
=> -n + 183 = 248d => n = 183 - 248d (vì n >0 => d<=0 và d nguyên ) 
=> n = 183 - 248d (với d là số nguyên <=0) 
vì n có 3 chữ số lớn nhất => n<=999 => d>= -3 => d = -3 
=> n = 927

16 tháng 4 2016

n=927

k nha

AH
Akai Haruma
Giáo viên
30 tháng 4 2023

Bài 1:
$(y+\frac{1}{3})+(y+\frac{1}{9})+(y+\frac{1}{27})+(y+\frac{1}{81})=\frac{56}{81}$

$(y+y+y+y)+(\frac{1}{3}+\frac{1}{9}+\frac{1}{27}+\frac{1}{81})=\frac{56}{81}$
$4\times y+\frac{40}{81}=\frac{56}{81}$

$4\times y=\frac{56}{81}-\frac{40}{81}=\frac{16}{81}$
$y=\frac{16}{81}:4=\frac{4}{81}$

AH
Akai Haruma
Giáo viên
30 tháng 4 2023

Bài 2:

$18: \frac{x\times 0,4+0,32}{x}+5=14$

$18: \frac{x\times 0,4+0,32}{x}=14-5=9$

$\frac{x\times 0,4+0,32}{x}=18:9=2$

$x\times 0,4+0,32=2\times x$

$2\times x-x\times 0,4=0,32$

$x\times (2-0,4)=0,32$
$x\times 1,6=0,32$
$x=0,32:1,6=0,2$

18 tháng 8 2016

\(\left(y-\frac{1}{2}\right):\left(\frac{1}{1.2}+\frac{1}{2.3}+\frac{1}{3.4}+\frac{1}{4.5}+...+\frac{1}{9.10}\right)=\frac{1}{3}\)

\(\Leftrightarrow\left(y-\frac{1}{2}\right):\left(1-\frac{1}{2}+\frac{1}{2}-\frac{1}{3}+\frac{1}{3}-\frac{1}{4}+\frac{1}{4}-...+\frac{1}{9}-\frac{1}{10}\right)=\frac{1}{3}\)

\(\Leftrightarrow\left(y-\frac{1}{3}\right):\left(1-\frac{1}{10}\right)=\frac{1}{3}\)

\(\Leftrightarrow\left(y-\frac{1}{2}\right):\frac{9}{10}=\frac{1}{3}\)

\(\Leftrightarrow\left(y-\frac{1}{2}\right)=\frac{3}{10}\)

\(\Leftrightarrow y=\frac{4}{5}\)

15 tháng 2 2017

4/5 nha bạn

19 tháng 5 2018

Gợi ý: Các biểu thức mũ chẵn đều không âm.

\(a^{2n}+b^{2n}\le0\Leftrightarrow a^{2n}+b^{2n}=0\Leftrightarrow a=b=0\)

19 tháng 5 2018

a,\(\left(x-\frac{2}{5}\right)^{2010}+\left(y+\frac{3}{7}\right)^{468}\)< \(0\)

Vì \(\left(x-\frac{2}{5}\right)^{2010}\);\(\left(y+\frac{3}{7}\right)^{468}\)đều > \(0\)

=> \(\left(x-\frac{2}{5}\right)^{2010}=0\)

     \(\left(y+\frac{3}{7}\right)^{468}=0\)

=> \(\left(x-\frac{2}{5}\right)^{2010}=0^{2010}\)

     \(\left(y+\frac{3}{7}\right)^{468}=0^{468}\)

=> \(x-\frac{2}{5}=0\)

      \(y-\frac{3}{7}=0\)

=> \(x=\frac{2}{5}\)

      \(y=\frac{3}{7}\)

Vậy \(x=\frac{2}{5}\)\(y=\frac{3}{7}\)